Docsity
Docsity

Prepare for your exams
Prepare for your exams

Study with the several resources on Docsity


Earn points to download
Earn points to download

Earn points by helping other students or get them with a premium plan


Guidelines and tips
Guidelines and tips

Seminar pdf 3? Idkkk, Lecture notes of Mathematics

Seminar pdf Simon Fraser university

Typology: Lecture notes

2024/2025

Uploaded on 06/17/2025

idk-asf
idk-asf 🇨🇦

1 document

1 / 5

Toggle sidebar

This page cannot be seen from the preview

Don't miss anything!

bg1
MATH 150: SEMINAR # 3 QUE ST ION S SOLUTIONS
Content:
Section 2.3: Limit Laws
Section 2.5: Continuity
Questions:
1. Is there a number bsuch that
lim
x2
bx210x+ 10 + b
x2x2
exists? If so, find the value of band the value of the limit.
Solution. We are interested in what happens near x= 2. In the denominator, we get:
(2)2(2) 2=422=0,
so in order for the limit to exist, the numerator must also be 0at x= 2. If this is not the case, then as the
limit approaches 2 from one side, the denominator gets smaller and smaller and the limit approaches
, while from the other side, the limit approaches −∞. This means that bmust be such that
b(2)210(2) + 10 + b= 4b20 + 10 + b= 5b10 = 0
which gives that b= 2. The limit is therefore,
lim
x2
2x210x+ 12
x2x2= lim
x2
2(x25x+ 6)
(x2)(x+ 1) = lim
x2
2
(x2)(x3)
(x2)(x+ 1) = lim
x2
2(x3)
x+ 1 =2
3
2. Suppose that
x4< f(x)< x2
if |x|<1and
x2< f(x)< x4
if |x|>1. Do the following limits exist? If so, find their values.
(a) lim
x→−1f(x)
(b) lim
x1f(x)
(c) lim
x0f(x)
Solution.
(a) We have the following limits:
lim
x→−1+x4= 1; lim
x→−1
x4= 1; lim
x→−1+x2= 1; lim
x→−1
x2= 1
so by application of the Squeeze Theorem we get that
lim
x→−1f(x) = 1.
(b) We have the following limits:
lim
x1+x4= 1; lim
x1
x4= 1; lim
x1+x2= 1; lim
x1
x2= 1
so by application of the Squeeze Theorem we get that
lim
x1f(x)=1.
pf3
pf4
pf5

Partial preview of the text

Download Seminar pdf 3? Idkkk and more Lecture notes Mathematics in PDF only on Docsity!

Content:

Section 2.3: Limit Laws

Section 2.5: Continuity

Questions:

  1. Is there a number b such that lim x→ 2

bx^2 − 10 x + 10 + b x^2 − x − 2 exists? If so, find the value of b and the value of the limit. Solution. We are interested in what happens near x = 2. In the denominator, we get:

(2)^2 − (2) − 2 = 4 − 2 − 2 = 0,

so in order for the limit to exist, the numerator must also be 0 at x = 2. If this is not the case, then as the limit approaches 2 from one side, the denominator gets smaller and smaller and the limit approaches ∞, while from the other side, the limit approaches −∞. This means that b must be such that

b(2)^2 − 10(2) + 10 + b = 4b − 20 + 10 + b = 5b − 10 = 0

which gives that b = 2. The limit is therefore,

lim x→ 2

2 x^2 − 10 x + 12 x^2 − x − 2

= lim x→ 2

2(x^2 − 5 x + 6) (x − 2)(x + 1)

= lim x→ 2

(^2) (x − 2)(x − 3)  (x − 2)(x + 1) = lim x→ 2

2(x − 3) x + 1

  1. Suppose that x^4 < f (x) < x^2 if |x| < 1 and x^2 < f (x) < x^4 if |x| > 1. Do the following limits exist? If so, find their values.

(a) lim x→− 1 f (x)

(b) lim x→ 1 f (x)

(c) lim x→ 0 f (x)

Solution.

(a) We have the following limits:

lim x→− 1 +

x^4 = 1; lim x→− 1 −

x^4 = 1; lim x→− 1 +

x^2 = 1; lim x→− 1 −

x^2 = 1

so by application of the Squeeze Theorem we get that

lim x→− 1 f (x) = 1.

(b) We have the following limits:

lim x→ 1 +^

x^4 = 1; lim x→ 1 −^

x^4 = 1; lim x→ 1 +^

x^2 = 1; lim x→ 1 −^

x^2 = 1

so by application of the Squeeze Theorem we get that

lim x→ 1 f (x) = 1.

(c) We have the following limits: lim x→ 0 x^4 = 0; lim x→ 0 x^2 = 0;

so by application of the Squeeze Theorem we get

lim x→ 0

f (x) = 0.

  1. Evaluate the following limits using limit theorems.

(a) lim x→ 10

x^2 − 100 x − 9 (b) lim x→ 10

f (x), where f (x) = x^2 for all x ̸= 10, but f (10) = 99.

(c) lim x→ 8

√ (^3) x − 2

x − 8

(d) lim x→ 8

(x − 8)(x + 2) |x − 8 | Solution.

(a) lim x→ 10

x^2 − 100 x − 9

(b) lim x→ 10 f (x) = lim x→ 10 x^2 = 100 so lim x→ 10 f (x) = 100

(c)

lim x→ 8

√ (^3) x − 2

x − 8 = lim x→ 8

x − 2)  ( √^3 x − 2)( √^3 x^2 + 2 √^3 x + 4) = lim^ x→^8

√ (^3) x (^2) + 2 √ (^3) x + 4 =

(d) For finding lim x→ 8

(x − 8)(x + 2) |x − 8 |

, we use

|x − 8 | =

x − 8 if x ≥ 8 −(x − 8) if x < 8.

Let’s consider left and right hand limits.

lim x→ 8 +^

(x − 8)(x + 2) (x^ −^ 8)^

= lim x→ 8 +

(x + 2) = 10.

lim x→ 8 −^

(x − 8)(x + 2) −(x − 8)

= lim x→ 8 −^

−(x + 2) = − 10.

Since the right and left hand limits are not equal, this limit does not exist.

  1. Use the Squeeze Theorem to show that

lim x→ 0

p x^3 + x^2 sin

π x

Solution. Let f (x) = −

x^3 + x^2 , g(x) =

x^3 + x^2 sin(π/x), and h(x) =

x^3 + x^2. Notice the domain of f (x) and h(x) is {x ∈ R|x ≥ − 1 }, i.e. those values such that x^3 + x^2 ≥ 0. Then we know that − 1 ≤ sin(π/x) ≤ 1 and so the following holds for x ≥ − 1 :

p x^3 + x^2 ≤

p x^3 + x^2 sin (π/x) ≤

p x^3 + x^2 ⇒ f (x) ≤ g(x) ≤ h(x)

And since lim x→ 0 f (x) = lim x→ 0 h(x) = 0, by the Squeeze Theorem, lim x→ 0 g(x) = 0.

which means that f is continuous from the left at x = 1 for any value of b. But it also has to be continuous from the right. We find the right hand limit:

lim x→ 1 +

f (x) = lim x→ 1 +

x^2 + 2b = 1 + 2b

In order for this function to be continuous from the right, we need limx→ 1 +^ f (x) = f (1), which means that b − 1 = 1 + 2b ⇒ b = − 2. Thus, when b = − 2 , f is continuous at x = 1. □

  1. Give one example of a function f (x) that is continuous for all values of x except x = 3, where it has a removable discontinuity. Explain how you know that f is discontinuous at x = 3, and how you know that the discontinuity is removable. Solution. Consider, for example, the function

f (x) =

 (^) x (^2) − 9 x− 3 if^ x^ ̸= 3 0 if x = 3.

Observe that, for x ̸= 3,

f (x) =

x^2 − 9 x − 3

(x + 3)(x − 3) x − 3

(x + 3)(x − 3)  (x − 3) =^ x^ + 3.

and conclude:

(a) The function f is continuous for on the set(−∞, 3) ∪ (3, ∞). (b) From lim x→ 3 f (x) = lim x→ 3 (x + 3) = 6 ̸= 0 = f (3)

it follows that the function f is not continuous at x = 3.

The graph of this function:

x

y

  1. If f (x) = x^2 + 10 sin x, show that there is a number c such that f (c) = 1000.

Solution. Observe that the given function if continuous on R. Also, for all x ∈ R,

− 1 ≤ sin x ≤ 1 ⇒ x^2 − 10 ≤ f (x) ≤ x^2 + 10

Since

10 ≈ 10 · 3 .15 = 31. 5 , we consider an interval that contains the number 31. 5. To keep our calculations simple, take the interval [0, 40]. Now

f (0) = 0^2 + 10 sin(0) = 0 < 1000 and f (40) = 40^2 + 10 sin(40) ≤ 1600 − 10 = 1590 > 1000.

Therefore, since f is continuous on the closed interval [0, 40] and since 1000 ∈ [f (0), f (40)] = [0, 1590], by the Intermediate Value Theorem there is a number c in (0, 40) such that f (c) = 1000.

  1. (a) Use the Intermediate Value Theorem to show that 2 x^ = (^10) x for some x > 0.

(b) Show that the equation 2 x^ = (^10) x has no solution for x < 0. Solution. Let f (x) = 2x^ − (^10) x. Note that the domain of f is the set R{ 0 } and that on its domain, as a sum of two continuous functions, f is continuous.

(a) Since f is continuous on (0, ∞) and since lim x→ 0 +

f (x) = −∞ and lim x→∞ f (x) = ∞, but the Intermediate Value Theorem there is a ∈ (0, ∞) such that f (a) = 0. (b) For all x ∈ (−∞, 0) we have that (^10) x < 0 which implies that for all x ∈ (−∞, 0) we have that all f (x) > 0.

  1. Suppose f is continuous on [1, 5] and the only solutions of the equation f (x) = 6 are x = 1 and x = 4. If f (2) = 8, explain why f (3) > 6. Solution. Suppose that f (3) < 6. By the Intermediate Value Theorem applied to the continuous function f on the closed interval [2, 3], the fact that f (2) = 8 > 6 and f (3) < 6 implies that there is a number c in (2, 3) such that f (c) = 6. This contradicts the fact that the only solutions of the equation f (x) = 6 are x = 1 and x = 4. Hence, our supposition that f (3) < 6 was incorrect. It follows that f (3) ≥ 6. But f (3) ̸= 6 because the only solutions of f (x) = 6 are x = 1 and x = 4. Therefore, f (3) > 6.